Hw 9 Chap 12

¡Supera tus tareas y exámenes ahora con Quizwiz!

A monopoly results in productive inefficiency because at the profit-maximizing output level,

ATC is not at its minimum level.

The economic incentive for price discrimination is based upon

The economic incentive for price discrimination is based upon

To maximize profit, a pure monopolist must

maximize the difference between total revenue and total cost.

If the variable costs of a profit-maximizing pure monopolist decline, the firm should

produce more output and charge a lower price.

The demand curve confronting a nondiscriminating pure monopolist is

the same as the industry's demand curve.

Suppose that a pure monopolist can sell 4 units of output at $2 per unit and 5 units at $1.75 per unit. The monopolist will produce and sell the fifth unit if its marginal cost is

$.75 or less.

Check my workCheck My Work button is now disabled1 Item 29 Item 29 2.5 points Total OutputPriceMarginal RevenueAverage Total CostMarginal Cost1$100$100$100.00$302908063.00263806052.67324704049.50405602049.6050650050.0052740-2052.2966830-4055.7580920-6060.671001010-8067.60130 Refer to the data for a nondiscriminating monopolist. At its profit-maximizing output, this firm's total profit will be

$82.

OutputPriceTotal Cost0$500$25013002602250290320035041505005100680 Refer to the demand and cost data for a pure monopolist given in the table. If the monopolist perfectly price-discriminated and sold each unit of the product at the maximum price the buyer of that unit would be willing to pay, and if the monopolist sold 4 units, then total revenue would be

$900.

If the industry depicted in this graph were purely competitive, the output quantity would be

160.

Which is a major criticism of a monopoly as a source of allocative inefficiency?

A monopolist fails to expand output to the level where the consumers' valuation of an additional unit is just equal to its opportunity cost.

Refer to the graphs of D and MR for a monopolist. Which of the following statements is true?

A price cut from P1 to P2 would lead to an increase in the amount of dollars consumers spend on the product.

In the graph, what is the profit-maximizing level of output for this pure monopolist?

B

If profits are maximized (or losses minimized), which of the following conditions is common to both unregulated monopoly and pure competition?

MR = MC

Refer to the provided graph for an industry. If the industry was initially a monopoly, but the monopolist was broken up into a large number of small, purely competitive firms and production cost-curves remained unchanged, then market price and industry output would be

P2 and Q2.

Refer to the cost and demand data for a pure monopolist. Suppose that this monopoly is subjected to a regulatory commission. If the commission seeks to achieve the most efficient allocation of resources for this industry, it should set the socially optimal price at

P2.

Refer to the diagram for a pure monopolist. If a regulatory commission seeks to achieve the socially optimal allocation of resources to this line of production, it will set a price of

P2.

If the industry depicted in this graph were served by a pure monopoly, the price and output quantity would be

P3 and Q1.

Refer to the graph for a pure monopoly. If the government regulated the monopoly and made the firm set a fair-return price, what price and quantity levels would we observe in the short run?

P3 and Q2

Which is true of a price-discriminating pure monopolist?

Profit will be higher than in the nondiscriminating case.

Refer to the graph, which shows a linear demand curve for a monopolist. Which of the following statements is correct?

The area 0QVS is greater than the area 0RWT.

Pure monopoly refers to

a single firm producing a product for which there are no close substitutes.

A price-discriminating monopolist will follow a system where

buyers with inelastic demand are charged higher prices than buyers with elastic demand.

In response to a cost-reducing technological breakthrough in the production of its product, a profit-maximizing monopolist will normally

decrease the price it charges for its product.

A pure monopoly firm will never charge a price in the inelastic range of its demand curve because lowering price to get into this region will

decrease total revenue, increase total cost, and decrease profit.

Which of the following is incorrect? Imperfectly competitive producers

do not compete with one another

Refer to the graph, which shows a total revenue curve for a monopolist. The firm's marginal revenue curve must be

downsloping.

Refer to the diagrams. The demand for Firm B's product is

elastic for prices above $4 and inelastic for prices below $4.

Under pure monopoly, a profit-maximizing firm will produce

in the elastic range of its demand curve.

The problem with adopting a fair-return pricing policy for a natural monopoly is that

it is not allocatively efficient.

If a pure monopolist is producing more output than the MR = MC output,

it will be in the interest of the firm, but not necessarily of society, to reduce output.

In many large U.S. cities, taxicab companies operate as near monopolies because of

licenses.

In the short-run equilibrium, a monopolist's profits

may be positive, negative, or zero.

There is some evidence to suggest that X-inefficiency is

more likely to occur in monopolistic firms than in competitive firms.

Large minimum efficient scale of plant combined with limited market demand may lead to

natural monopoly.

A market where there are many firms, but one firm dominates and has the bulk (85 percent) of sales in the market, is called a

near-monopoly.

When the value of a product to each user, including existing users, increases due to an increase in the total number of users—as in the case of Facebook—we refer to this as

network effects.

Refer to the diagram for a nondiscriminating monopolist. The profit-seeking monopolist will

never produce an output larger than q2.

Answer the question on the basis of the provided demand and cost data for a pure monopolist. Demand DataCost DataPriceQuantity DemandedOutputTotal Cost$5.5033$5.005.00446.004.50556.503.85667.503.35779.002.908811.002.509914.00 The profit-maximizing monopolist will realize a

profit of $16.

One argument for having the government regulate natural monopolies is that without regulation,

these monopolies produce at a level where marginal benefit is greater than marginal cost.

A pure monopolist's short-run profit-maximizing or loss-minimizing position is such that price

will vertically intersect demand where MR = MC.


Conjuntos de estudio relacionados

Speak Up! Chapter 10 (Key Terms and Review Questions)

View Set

Chapter 18 Positive Punishment Procedures and the Ethics of Punishment

View Set

Unit 4 Estate Planning FINA 4397

View Set

nouns, pronouns, adjective, and verbs

View Set